aboly

New Member
ارسال ها
266
لایک ها
110
امتیاز
0
#61

msaeids

New Member
ارسال ها
83
لایک ها
44
امتیاز
0
#62
پاسخ : .::ماراتن 4 درس ::.

ولی قرار شد که هرکی سوال میذاره با بقیه سوالها همراه باشه
که دیگه سوال گم نشه
الان کسی که میدونه چه خبره همه ی سوالا رو بذاره تو یه پست:4:
ممنون:4:
و ببخشید برای این اسپم
 

nima1376

New Member
ارسال ها
218
لایک ها
93
امتیاز
0
#63
پاسخ : .::ماراتن 4 درس ::.

سوالای ماراتن:
(1 جبر)
اگر

و

نشان دهید

(2 جبر)فرض کنید
و دارای n ریشه صحیح متمایز باشد . ثابت کنید
عاملی با درجه دست کم
دارد که در
تحویل ناپذیر است .

(1 نظریه) تعداد چهار تایی های x,y,z,w را از اعداد حسابی بیابید x,y,z,w کوچکترمساوی 36
و

x[SUP]2[/SUP]+y[SUP]2[/SUP]به همنهشت 37 برابر با z[SUP]3[/SUP]+w[SUP]3[/SUP]

باشد

راهنمایی(جواب عدد کوچکی نیست)

(2 نظریه)ثابت کنید در بسط اعشاری
بین رقم یک میلیونیم تا 3 میلینیم حداقل یک رقم ناصفر وجود دارد .


(2 هندسه)
وسط
از مثلث
است و داریم :
. دایره محاطی مثلث میانه
را در
قطع میکند . نشان دهید ​


(1 هندسه) در مثلث abc از h دو خط عمود برهم
میکشیم تا abوac را درeوf قطع کند.
ef دایره به قطر bc را درm قطع کند.
نشان دهید cm موازی he است.

(1 ترکیبیات)
مربعی به ضلع 1 داریم
مثلثی درون آن است که مرکز مربع خارج آن است.
ثابت کنید ضلعی از مثلث کمتر از 1 است.

به علت هماهنگ نبودن من الان دوتا از این سطح داریم
اینم دومین ترکیبیات 1:
یه سوال خیلی کلاسیک و قدیمی
فردی دور یک دایره به محیط یک راه میرودبا طول قدم
که عددی گنگ است
روی این دایره گودالی به قطر
داریم ثابت کنید قطر گودال هر چه کوچک باشد شخص در آن میافتد
بیانی دیگر
ثابت کنید برای هر عدد گنگ
و عدد حقیقی ومثبت
عدد طبیعی
موجود است که

ب) به مرکز هریک از نقاط شبکه ای دایره ای به شعاع
رده ایم
ثابت کنید مثلث متساوی الاضلاعی داریم که رووسش داخل این دایره ها باشند

(2 ترکیبیات)
اعداد
تا
در یک جدول10*10 چیده شده اند
در هر مرحله میتوانیم جای هر دو عددی را که میخواهیم با هم عوض کنیم
ثابت کنید میتوان
حرکت انجام داد به گونه ای که مجموع اعداد واقع در خانه های مجاور عددی مرکب باشد

لطفا هر کسی سوال میذاره همین پست رو نقل قول کنه
مواردش رو کپی کنه سوالش رو هم تو جاش جایگزین کنه
ممنون

 

aboly

New Member
ارسال ها
266
لایک ها
110
امتیاز
0
#64
پاسخ : .::ماراتن 4 درس ::.

برای یک جبر

درابتدا همه اعداد را برابر با توان 2 عددی دیگر بگیرید(یعنی a =x[SUP]2[/SUP]
(

سپس تابع محدب بسازید

حال باید ثابت کنید : x[SUP]2[/SUP]y[SUP]2[/SUP]+x[SUP]2[/SUP]z[SUP]2[/SUP]+y[SUP]2[/SUP]z[SUP]2[/SUP]+3 از x+y+z کوچکتر است

که این با لاگرانژ ثابت می شود

​اگه درسته سوال جدید بذارم
 

nima1376

New Member
ارسال ها
218
لایک ها
93
امتیاز
0
#65
پاسخ : .::ماراتن 4 درس ::.

برای یک جبر

درابتدا همه اعداد را برابر با توان 2 عددی دیگر بگیرید(یعنی a =x[SUP]2[/SUP]
(

سپس تابع محدب بسازید

حال باید ثابت کنید : x[SUP]2[/SUP]y[SUP]2[/SUP]+x[SUP]2[/SUP]z[SUP]2[/SUP]+y[SUP]2[/SUP]z[SUP]2[/SUP]+3 از x+y+z کوچکتر است

که این با لاگرانژ ثابت می شود

​اگه درسته سوال جدید بذارم
از اینکه

چه استفاده ای میکنی؟
بله هرسوال نامساوی با لاگرانژ حل میشه ولی راه حل خلاقیتی مهم است.
اگر میشه راهی به جز لاگرانژ پیشنهاد بدید.
به نظر من این راه هیچ ارزشی ندارد.
 

math

New Member
ارسال ها
1,129
لایک ها
1,096
امتیاز
0
#66
پاسخ : .::ماراتن 4 درس ::.

از اینکه

چه استفاده ای میکنی؟
بله هرسوال نامساوی با لاگرانژ حل میشه ولی راه حل خلاقیتی مهم است.
اگر میشه راهی به جز لاگرانژ پیشنهاد بدید.
به نظر من این راه هیچ ارزشی ندارد.

البته ( این نظر شخصی منه ) حل سوال با سواد و کم یا نبودن خلاقیت هیچ کم از راه حل خلاقانه که ندارد هیچ شاید بالا تر هم باشد

و یه نکته دیگه این که تمام نامساوی ها با لاگرانژ حل نمیشه

بهتره هر سوال رو با سواد سطحش حل کنیم اگر این سوال سطح دو بود راه حل ایشون کاملا درست و ... بود اما این سوال در سطح یک اورده شده پس باید با سواد مرحله حل شود
 

nima1376

New Member
ارسال ها
218
لایک ها
93
امتیاز
0
#67
پاسخ : .::ماراتن 4 درس ::.

البته ( این نظر شخصی منه ) حل سوال با سواد و کم یا نبودن خلاقیت هیچ کم از راه حل خلاقانه که ندارد هیچ شاید بالا تر هم باشد

و یه نکته دیگه این که تمام نامساوی ها با لاگرانژ حل نمیشه

بهتره هر سوال رو با سواد سطحش حل کنیم اگر این سوال سطح دو بود راه حل ایشون کاملا درست و ... بود اما این سوال در سطح یک اورده شده پس باید با سواد مرحله حل شود
هرکسی نظری داره.
 
ارسال ها
337
لایک ها
82
امتیاز
0
#68
پاسخ : .::ماراتن 4 درس ::.

در اعداد طبیعیه

این سوال n 4 شورت لیست 2012 هستش
سوالتون با حالت بندی روی m.n هم حل میشه فقط یه خورده دهن سرویسی داره:155:
 

aboly

New Member
ارسال ها
266
لایک ها
110
امتیاز
0
#69
پاسخ : .::ماراتن 4 درس ::.

از اینکه

چه استفاده ای میکنی؟
بله هرسوال نامساوی با لاگرانژ حل میشه ولی راه حل خلاقیتی مهم است.
اگر میشه راهی به جز لاگرانژ پیشنهاد بدید.
به نظر من این راه هیچ ارزشی ندارد.
خوب این شرط تو لاگرانژ استفاده میشه دیگه

اگه با یه راه دیگه بلدی اسم نامساوی که استفاده کردی رو بگو تا با همون حل کنم (به عنوان راهنمایی بگو)

:176::176::176:
 

math

New Member
ارسال ها
1,129
لایک ها
1,096
امتیاز
0
#70
پاسخ : .::ماراتن 4 درس ::.

(1 نظریه) تعداد چهار تایی های x,y,z,w را از اعداد حسابی بیابید x,y,z,w کوچکترمساوی 36
و

x[SUP]2[/SUP]+y[SUP]2[/SUP]به همنهشت 37 برابر با z[SUP]3[/SUP]+w[SUP]3[/SUP]

باشد

سوال سطح 1 نظریه اعداد یعنی سوال سطح مرحله 2 نه مرحله 1 ;)

من سوال دیگه ای میذارم :

ثابت کنید در دنباله ی زیر بی نهایت عامل اول وجود دارد
( بدون استفاده از قضیه کوبایاشی البته ;) )
 
آخرین ویرایش توسط مدیر

sa1378

New Member
ارسال ها
1,403
لایک ها
1,077
امتیاز
0
#71
پاسخ : .::ماراتن 4 درس ::.

سوال سطح 1 نظریه اعداد یعنی سوال سطح مرحله 2 نه مرحله 1 ;)

من سوال دیگه ای میذارم :

ثابت کنید در دنباله ی زیر بی نهایت عامل اول وجود دارد
( بدون استفاده از قضیه کوبایاشی البته ;) )
برای مرحله یک هم بزارین
اصلا بهتره بکنینش 3 سطح
 

math

New Member
ارسال ها
1,129
لایک ها
1,096
امتیاز
0
#72
پاسخ : .::ماراتن 4 درس ::.

برای مرحله یک هم بزارین
اصلا بهتره بکنینش 3 سطح

خوب میشه برای اون یه تاپیک دیگه زد چون خیلی مرحله 2و3 یا مرحله 1 فرق داره
 

msaeids

New Member
ارسال ها
83
لایک ها
44
امتیاز
0
#73
پاسخ : .::ماراتن 4 درس ::.

سوال سطح 1 نظریه اعداد یعنی سوال سطح مرحله 2 نه مرحله 1 ;)

من سوال دیگه ای میذارم :

ثابت کنید در دنباله ی زیر بی نهایت عامل اول وجود دارد
( بدون استفاده از قضیه کوبایاشی البته ;) )
شما به این سوال مطمینید؟؟؟
اگر m که مقدار ثابت هست رو برابر 1 فرض کنیم اوننوقت باید ثابت کنیم بینهایت عدد اول فرما داریم؟؟؟
که هنوز ثابت نشده؟؟؟
آیا من در اشتباهم؟؟؟
 

math

New Member
ارسال ها
1,129
لایک ها
1,096
امتیاز
0
#74
پاسخ : .::ماراتن 4 درس ::.

شما به این سوال مطمینید؟؟؟
اگر m که مقدار ثابت هست رو برابر 1 فرض کنیم اوننوقت باید ثابت کنیم بینهایت عدد اول فرما داریم؟؟؟
که هنوز ثابت نشده؟؟؟
آیا من در اشتباهم؟؟؟

سوال نگفته که بی نهایت عدد اول توی این دنباله هست گفته بی نهایت عامل اول هست یعنی بی نهایت عدد اول وجود دارد که یکی از اعضای دنباله بر ان بخش پذیر شود
 

aboly

New Member
ارسال ها
266
لایک ها
110
امتیاز
0
#75
پاسخ : .::ماراتن 4 درس ::.

سوال سطح 1 نظریه اعداد یعنی سوال سطح مرحله 2 نه مرحله 1 ;)

من سوال دیگه ای میذارم :

ثابت کنید در دنباله ی زیر بی نهایت عامل اول وجود دارد
( بدون استفاده از قضیه کوبایاشی البته ;) )

سوالی که گذاشتم مال ترکیه بوده بعد تازه نیاز به دقت خیلی زیادی داره اگه واقعا آسونه پس جواب دقیقش رو بنویس(فرض کن همین سوال 1 مرحله 2 باشه )
 

nima1376

New Member
ارسال ها
218
لایک ها
93
امتیاز
0
#76
پاسخ : .::ماراتن 4 درس ::.

از کسانی که سوالات رو قرار دادند اگر بعد مدتی سوال حل نشد جوابشو قرار بدن.
 

math

New Member
ارسال ها
1,129
لایک ها
1,096
امتیاز
0
#77
پاسخ : .::ماراتن 4 درس ::.

سوالی که گذاشتم مال ترکیه بوده بعد تازه نیاز به دقت خیلی زیادی داره اگه واقعا آسونه پس جواب دقیقش رو بنویس(فرض کن همین سوال 1 مرحله 2 باشه )

ببخشید من بلد نبودم حساب کنم سوال جدید گذاشتم ولی یک چیزی این سوال سبک مرحله 2 ای نداره :1:


سطح 2 (مرحله 3 )

فرض کنید
و دارای n ریشه صحیح متمایز باشد . ثابت کنید
عاملی با درجه دست کم
دارد که در
تحویل ناپذیر است .
خوب مثل اینکه جبر کاری نداشتیم که سوال 2 رو حل کنه من راه حلش رو میزارم :

فرض کنید
دارای k عامل باشد پس داریم :




حال به وضوح درجه S ها زوج است

حال فرض کنید ریشه ها
باشد پس به وضوح داریم :




حال اگر از رابطه ی اول مشتق بگیرید و به این نکته توجه کنید که :






حال اگر جای x ریشه ها را بگذاریم میدانیم :





پس معقول هست که چند جمله ایه روبرو را در نظر بگیریم :




حال میدانیم :





که در ان
پس حداقل یکی از S ها در جه بزرگ تر از
دارد ( در دو حالت زوج و فرد بررسی کنید و برهان خلف بزنید )

سوال جدید هم تا شب میزارم ببخشید

 

msaeids

New Member
ارسال ها
83
لایک ها
44
امتیاز
0
#78
پاسخ : .::ماراتن 4 درس ::.

سوالای ماراتن:
(1 جبر)
اگر

و

نشان دهید

(2 جبر)نداریم!

(1 نظریه) ثابت کنید در دنباله ی زیر بی نهایت عامل اول وجود دارد
( بدون استفاده از قضیه کوبایاشی البته ;) )

(2 نظریه)ثابت کنید در بسط اعشاری
بین رقم یک میلیونیم تا 3 میلینیم حداقل یک رقم ناصفر وجود دارد .


(2 هندسه)
وسط
از مثلث
است و داریم :
. دایره محاطی مثلث میانه
را در
قطع میکند . نشان دهید ​


(1 هندسه) در مثلث abc از h دو خط عمود برهم
میکشیم تا abوac را درeوf قطع کند.
ef دایره به قطر bc را درm قطع کند.
نشان دهید cm موازی he است.

(1 ترکیبیات)
مربعی به ضلع 1 داریم
مثلثی درون آن است که مرکز مربع خارج آن است.
ثابت کنید ضلعی از مثلث کمتر از 1 است.


(2 ترکیبیات)
یک جدول 2013*1392 داریم که بیش از 2011*1390 خانه ی آن سفیدند
در هر مرحله میتوانیم یک جدول 2*2 از آن را که سه خانه ی سیاه دارد در نظر بگیریم و خانه ی چهارم رو هم سیاه کنیم
ثابت کنید با انجام این عمل نمیتوان همه ی خانه های جدول را سیاه کرد.

لطفا هر کسی سوال میذاره همین پست رو نقل قول کنه

مواردش رو کپی کنه سوالش رو هم تو جاش جایگزین کنه
ممنون




من از جناب math درخواست دارم جواب سوال سطح 2 نظریه رو هم بذارن
منم جواب سوال ترکیبیات هایی که خودم گذاشتم رو میذارم

سوال اول:(
یه سوال خیلی کلاسیک و قدیمی​
فردی دور یک دایره به محیط یک راه میرودبا طول قدم
که عددی گنگ است
روی این دایره گودالی به قطر
داریم ثابت کنید قطر گودال هر چه کوچک باشد شخص در آن میافتد
بیانی دیگر
ثابت کنید برای هر عدد گنگ
و عدد حقیقی ومثبت
عدد طبیعی
موجود است که

ب) به مرکز هریک از نقاط شبکه ای دایره ای به شعاع
رده ایم
ثابت کنید مثلث متساوی الاضلاعی داریم که رووسش داخل این دایره ها باشند)
جواب:
پاره خطی به طول
رو در نظر بگیرید.
اگر از وسطش یه عمود به طول
خارج کنیم راس سوم مثلث متساوی الاضلاع بدست می آید
حالا طبق بخش الف میدانیم
ای داریم که
پس راس سوم داخل یکی از دایره ها واقع میشود
سوال جایگزینش هم که از قبل وجود داشت
حالا هم جواب سوال دوم:(اعداد
تا
در یک جدول10*10 چیده شده اند
در هر مرحله میتوانیم جای هر دو عددی را که میخواهیم با هم عوض کنیم
ثابت کنید میتوان
حرکت انجام داد به گونه ای که مجموع اعداد واقع در خانه های مجاور عددی مرکب باشد)
جواب:جدول رو از وسط تا میکنیم تا دوتا جدول 5*10 به وجود بیاد
تو یکی از این جدول ها تعداد اعداد فرد کمتره
عدد های فرد این جدول رو با اعداد زوج جدول دیگری عوض کنید
حداکثر 25 حرکت لازمه
حالا مرز بین دو جدول دیگه جمع اعدادش زوج نیست
میتونیم با 10 حرکت (هر خونه یکبار )کاری بکنیم که مجموع خونه های مجاور مضرب3 بشود و به وضوح مضرب سه به تعداد کافی داریم
سوال جایگزین رو تو سوالای مراتن گذاشتم
 
لایک ها math

sa1378

New Member
ارسال ها
1,403
لایک ها
1,077
امتیاز
0
#79
پاسخ : .::ماراتن 4 درس ::.

یعنی قشنگ همه ثابت کنیده
جای من نیست اینجا
 

msaeids

New Member
ارسال ها
83
لایک ها
44
امتیاز
0
#80
پاسخ : .::ماراتن 4 درس ::.

یعنی قشنگ همه ثابت کنیده
جای من نیست اینجا
حرف شما رو قبول دارم
این یکی از اشکالات ماراتن هاست که ممکنه برای بعضیها سوالی رو بسوزونه
ولی خوب وقتی چند نفر به مدت چند روز روی یه سوال فکر میکنن و به نتیجه ی قابل قبولی نمیرسن باید جوابش رو گذاشت
شما هم اگه اخساس میکنین روی سوال کمتر از اندازه ای که باید فکر کردین خوب برید کامل روش فکر کنید و بعدا برگردید و جواب کاملش رو بخونید
لازم نیست که هرکی جواب رو گذاشت شما هم همون موقع جواب رو بخونید
 
بالا